A Return to Your Roots

Algebra Level 5

P ( x ) = 4 x 4 + j = 1 4 ( 5 j ) ( x 4 + j + x 4 j ) \large P(x)=4x^{4}+\sum_{j=1}^4 (5-j)(x^{4+j}+x^{4-j})

Let z 1 , z 2 , z 3 , , z k z_{1},z_{2},z_{3}, \ldots,z_{k} be the distinct roots of P ( x ) P(x) , and let z n = a n + b n i z_{n}=a_{n}+b_{n}i for n = 1 , 2 , 3 , , k n=1,2,3,\ldots,k , where i = 1 i=\sqrt{-1} and a n a_{n} and b n b_{n} are real numbers. Let

n = 1 k b n = m + p q \sum_{n=1}^k \left|b_{n}\right|=m+p\sqrt{q}\

where m m , p p and q q are positive integers and q q is not divisible by the square of any prime. Find m + p + q m+p+q .


Inspired by the 2003 AIME II Problem 15


The answer is 7.

This section requires Javascript.
You are seeing this because something didn't load right. We suggest you, (a) try refreshing the page, (b) enabling javascript if it is disabled on your browser and, finally, (c) loading the non-javascript version of this page . We're sorry about the hassle.

2 solutions

Brandon Monsen
Nov 3, 2015

The expansion of P ( x ) \large P(x) would be

P ( x ) = x 8 + 2 x 7 + 3 x 6 + 4 x 5 + 4 x 4 + 4 x 3 + 3 x 2 + 2 x + 1 P(x)=x^{8}+2x^{7}+3x^{6}+4x^{5}+4x^{4}+4x^{3}+3x^{2}+2x+1

This can be factored into

P ( x ) = ( x 5 + x 4 + x 3 + x 2 + x + 1 ) ( x 3 + x 2 + x + 1 ) P(x)=(x^{5}+x^{4}+x^{3}+x^{2}+x+1)(x^{3}+x^{2}+x+1)

We know that ( x 5 + x 4 + x 3 + x 2 + x + 1 ) ( x 1 ) = x 6 1 (x^{5}+x^{4}+x^{3}+x^{2}+x+1)(x-1)=x^{6}-1 and ( x 3 + x 2 + x + 1 ) ( x 1 ) = x 4 1 (x^{3}+x^{2}+x+1)(x-1)=x^{4}-1

so we can rewrite P ( x ) \large P(x) as:

P ( x ) = x 6 1 x 1 × x 4 1 x 1 P(x)=\frac{x^{6}-1}{x-1} \times \frac{x^{4}-1}{x-1} .

for P ( x ) \large P(x) to equal 0 0 , x 6 1 = 0 x^{6}-1=0 or x 4 1 = 0 x^{4}-1=0 , where x x is not equal to 1 1 .

This means we have all of the 6 t h 6^{th} and 4 t h 4^{th} roots of unity, exlcuding 1 1 itself.

We can rewrite 1 1 as

cos ( 0 + 2 π k ) + i sin ( 0 + 2 π k ) \cos (0+2\pi k) +i \sin (0+2\pi k)

where k k is an integer.

De Moivre's Theorem states that

( cos ( θ ) + i sin ( θ ) ) n = ( cos ( n θ ) + i sin ( n θ ) ) (\cos (\theta) +i \sin (\theta))^{n}=(\cos (n \theta) +i \sin (n \theta)) .

Using this, we can conclude that the sixth roots of unity are

1 1 6 = cos ( 1 6 ( 0 + 2 π k ) ) + i sin ( 1 6 ( 0 + 2 π k ) ) 1^{\frac{1}{6}}=\cos (\frac{1}{6}(0+2\pi k)) +i \sin (\frac{1}{6}(0+2\pi k))

which simplifies to

cos ( 1 3 π k ) + i sin ( 1 3 π k ) ) \cos (\frac{1}{3}\pi k) +i \sin (\frac{1}{3}\pi k)) .

Plugging in some values for k k will give us our distinct sixth roots of unity are

1 , 1 , 1 2 + 3 2 i , 1 2 3 2 i , 1 2 + 3 2 i , 1 2 3 2 i 1, -1, \frac{1}{2}+\frac{\sqrt{3}}{2}i,\frac{1}{2}-\frac{\sqrt{3}}{2}i,-\frac{1}{2}+\frac{\sqrt{3}}{2}i,-\frac{1}{2}-\frac{\sqrt{3}}{2}i .

We repeat the same process for our fourth roots, and get

1 1 4 = cos ( π 2 k ) + i sin ( π 2 k ) 1^{\frac{1}{4}}= \cos (\frac{\pi}{2}k)+i \sin (\frac{\pi}{2}k)

which, when plugging in some values for k k gets our distinct fourth roots of unity as

1 , 1 , i , i 1,-1,i,-i .

We can ignore 1 1 , since it is not in our domain, and 1 -1 is included twice, once as a fourth root and once as a sixth root. This means that all of our distinct roots are

1 , i , i , 1 2 + 3 2 i , 1 2 3 2 i , 1 2 + 3 2 i , 1 2 3 2 i -1, i, -i, \frac{1}{2}+\frac{\sqrt{3}}{2}i,\frac{1}{2}-\frac{\sqrt{3}}{2}i,-\frac{1}{2}+\frac{\sqrt{3}}{2}i,-\frac{1}{2}-\frac{\sqrt{3}}{2}i .

We were asked to find the sum of the magnitudes of the imaginary part of all of our distinct roots. This gives us

2 + 4 ( 3 2 ) 2+4(\frac{\sqrt{3}}{2})

which simplifies to

2 + 2 3 2+2\sqrt{3} .

Since that value satisfies the constraints of m , p , q m,p,q , our answer is

2 + 2 + 3 = 7 \large 2+2+3=\boxed{7}

Aareyan Manzoor
Nov 23, 2015

write as x 8 + 2 x 7 + 3 x 6 + 4 x 5 + 5 x 4 + 4 x 3 + 3 x 2 + 2 x + 1 x 4 x^8+2x^7+3x^6+4x^5+5x^4+4x^3+3x^2+2x+1-x^4 by the link provided, we can easily deduce this is equal to ( x 4 + x 3 + x 2 + x + 1 ) 2 x 4 (x^4+x^3+x^2+x+1)^2-x^4 apply basic algebra identity ( x 4 + x 3 + x + 1 ) ( x 4 + x 3 + x 2 + x 2 + x + 1 ) (x^4+x^3+x+1)(x^4+x^3+x^2+x^2+x+1) ( x 3 ( x + 1 ) + ( x + 1 ) ) ( x 2 ( x 2 + x + 1 ) + ( x 2 + x + 1 ) ) (x^3(x+1)+(x+1))(x^2(x^2+x+1)+(x^2+x+1)) ( x 3 + 1 ) ( x + 1 ) ( x 2 + 1 ) ( x 2 + x + 1 ) (x^3+1)(x+1)(x^2+1)(x^2+x+1) we have that ( x 3 + 1 ) = 0 x = 1 , 1 2 ± i 3 2 (x^3+1)=0\Longrightarrow x=-1,\dfrac{1}{2}\pm i\dfrac{\sqrt{3}}{2} x 2 + 1 = 0 x = ± i x^2+1=0\Longrightarrow x=\pm i x 2 + x + 1 = 0 x = 1 2 ± i 3 2 x^2+x+1=0\Longrightarrow x=-\dfrac{1}{2}\pm i\dfrac{\sqrt{3}}{2} x + 1 = 0 x = 1 x+1=0\Longrightarrow x=-1 the sum of the modulus to im part is 3 2 + 3 2 + 1 + 1 + 3 2 + 3 2 \dfrac{\sqrt{3}}{2}+\dfrac{\sqrt{3}}{2}+1+1+\dfrac{\sqrt{3}}{2}+\dfrac{\sqrt{3}}{2} = 2 + 2 3 =2+2\sqrt{3}

0 pending reports

×

Problem Loading...

Note Loading...

Set Loading...